Saltar al contenido principal
LibreTexts Español

8.4: Completitud y compacidad

  • Page ID
    111570
  • \( \newcommand{\vecs}[1]{\overset { \scriptstyle \rightharpoonup} {\mathbf{#1}} } \)

    \( \newcommand{\vecd}[1]{\overset{-\!-\!\rightharpoonup}{\vphantom{a}\smash {#1}}} \)

    \( \newcommand{\id}{\mathrm{id}}\) \( \newcommand{\Span}{\mathrm{span}}\)

    ( \newcommand{\kernel}{\mathrm{null}\,}\) \( \newcommand{\range}{\mathrm{range}\,}\)

    \( \newcommand{\RealPart}{\mathrm{Re}}\) \( \newcommand{\ImaginaryPart}{\mathrm{Im}}\)

    \( \newcommand{\Argument}{\mathrm{Arg}}\) \( \newcommand{\norm}[1]{\| #1 \|}\)

    \( \newcommand{\inner}[2]{\langle #1, #2 \rangle}\)

    \( \newcommand{\Span}{\mathrm{span}}\)

    \( \newcommand{\id}{\mathrm{id}}\)

    \( \newcommand{\Span}{\mathrm{span}}\)

    \( \newcommand{\kernel}{\mathrm{null}\,}\)

    \( \newcommand{\range}{\mathrm{range}\,}\)

    \( \newcommand{\RealPart}{\mathrm{Re}}\)

    \( \newcommand{\ImaginaryPart}{\mathrm{Im}}\)

    \( \newcommand{\Argument}{\mathrm{Arg}}\)

    \( \newcommand{\norm}[1]{\| #1 \|}\)

    \( \newcommand{\inner}[2]{\langle #1, #2 \rangle}\)

    \( \newcommand{\Span}{\mathrm{span}}\) \( \newcommand{\AA}{\unicode[.8,0]{x212B}}\)

    \( \newcommand{\vectorA}[1]{\vec{#1}}      % arrow\)

    \( \newcommand{\vectorAt}[1]{\vec{\text{#1}}}      % arrow\)

    \( \newcommand{\vectorB}[1]{\overset { \scriptstyle \rightharpoonup} {\mathbf{#1}} } \)

    \( \newcommand{\vectorC}[1]{\textbf{#1}} \)

    \( \newcommand{\vectorD}[1]{\overrightarrow{#1}} \)

    \( \newcommand{\vectorDt}[1]{\overrightarrow{\text{#1}}} \)

    \( \newcommand{\vectE}[1]{\overset{-\!-\!\rightharpoonup}{\vphantom{a}\smash{\mathbf {#1}}}} \)

    \( \newcommand{\vecs}[1]{\overset { \scriptstyle \rightharpoonup} {\mathbf{#1}} } \)

    \( \newcommand{\vecd}[1]{\overset{-\!-\!\rightharpoonup}{\vphantom{a}\smash {#1}}} \)

    Secuencias de Cauchy e integridad

    Al igual que con las secuencias de números reales podemos definir secuencias de Cauchy.

    Dejar\((X,d)\) ser un espacio métrico. Una secuencia\(\{ x_n \}\) en\(X\) es una secuencia Cauchy si por cada\(\epsilon > 0\) existe una\(M \in {\mathbb{N}}\) tal que para todos\(n \geq M\) y para todos\(k \geq M\) tenemos\[d(x_n, x_k) < \epsilon .\]

    La definición es otra vez simplemente una traducción del concepto de los números reales a los espacios métricos. Entonces una secuencia de números reales es Cauchy en el sentido de si y solo si es Cauchy en el sentido anterior, siempre que equipemos los números reales con la métrica estándar\(d(x,y) = \left\lvert {x-y} \right\rvert\).

    Dejar\((X,d)\) ser un espacio métrico. Decimos que\(X\) es completo o Cauchy-complete si cada secuencia de Cauchy\(\{ x_n \}\) en\(X\) converge en un\(x \in X\).

    El espacio\({\mathbb{R}}^n\) con la métrica estándar es un espacio métrico completo.

    Para\({\mathbb{R}}= {\mathbb{R}}^1\) ello se comprobó en.

    Tomar\(n > 1\). Dejemos\(\{ x^j \}_{j=1}^\infty\) ser una secuencia Cauchy en\({\mathbb{R}}^n\), donde escribimos\(x^j = \bigl(x_1^j,x_2^j,\ldots,x_n^j\bigr) \in {\mathbb{R}}^n\). Como la secuencia es Cauchy\(\epsilon > 0\), dada, existe\(M\) tal que por todos\(i,j \geq M\) tenemos\[d(x^i,x^j) < \epsilon.\]

    Arreglar algunos\(k=1,2,\ldots,n\), porque\(i,j \geq M\) tenemos\[\bigl\lvert x_k^i - x_k^j \bigr\rvert = \sqrt{{\bigl(x_k^i - x_k^j\bigr)}^2} \leq \sqrt{\sum_{\ell=1}^n {\bigl(x_\ell^i-x_\ell^j\bigr)}^2} = d(x^i,x^j) < \epsilon .\] De ahí que la secuencia\(\{ x_k^j \}_{j=1}^\infty\) sea Cauchy. Como\({\mathbb{R}}\) es completa la secuencia converge; existe\(x_k \in {\mathbb{R}}\) tal que existe\(x_k = \lim_{j\to\infty} x_k^j\).

    Escribir\(x = (x_1,x_2,\ldots,x_n) \in {\mathbb{R}}^n\). Por tenemos que\(\{ x^j \}\) converge hasta\(x \in {\mathbb{R}}^n\) y por lo tanto\({\mathbb{R}}^n\) está completo.

    Compacidad

    Dejar\((X,d)\) ser un espacio métrico y\(K \subset X\). El conjunto\(K\) se establece para ser compacto si para cualquier colección de conjuntos abiertos\(\{ U_{\lambda} \}_{\lambda \in I}\) tal que\[K \subset \bigcup_{\lambda \in I} U_\lambda ,\] existe un subconjunto finito\(\{ \lambda_1, \lambda_2,\ldots,\lambda_k \} \subset I\) tal que\[K \subset \bigcup_{j=1}^k U_{\lambda_j} .\]

    Se dice que una colección de conjuntos abiertos\(\{ U_{\lambda} \}_{\lambda \in I}\) como el anterior es una cubierta abierta de\(K\). Entonces, una manera de decir que\(K\) es compacto es decir que cada cubierta abierta de\(K\) tiene una subcubierta finita.

    Dejar\((X,d)\) ser un espacio métrico. Un conjunto compacto\(K \subset X\) está cerrado y acotado.

    Primero, demostramos que un conjunto compacto está acotado. Arreglar\(p \in X\). Tenemos la tapa abierta\[K \subset \bigcup_{n=1}^\infty B(p,n) = X .\] Si\(K\) es compacta, entonces existe algún conjunto de índices\(n_1 < n_2 < \ldots < n_k\) tal que\[K \subset \bigcup_{j=1}^k B(p,n_j) = B(p,n_k) .\] As\(K\) está contenido en una bola,\(K\) está acotado.

    A continuación, mostramos un conjunto que no está cerrado no es compacto. Supongamos que\(\overline{K} \not= K\), es decir, hay un punto\(x \in \overline{K} \setminus K\). Si\(y \not= x\), entonces para\(n\) con\(\nicefrac{1}{n} < d(x,y)\) nosotros tenemos\(y \notin C(x,\nicefrac{1}{n})\). Además\(x \notin K\), así\[K \subset \bigcup_{n=1}^\infty C(x,\nicefrac{1}{n})^c .\] como una bola cerrada está cerrada,\(C(x,\nicefrac{1}{n})^c\) está abierta, y así tenemos una cubierta abierta. Si tomamos alguna colección finita de índices\(n_1 < n_2 < \ldots < n_k\), entonces\[\bigcup_{j=1}^k C(x,\nicefrac{1}{n_j})^c = C(x,\nicefrac{1}{n_k})^c\] As\(x\) está en el cierre, tenemos\(C(x,\nicefrac{1}{n_k}) \cap K \not= \emptyset\), por lo que no hay una subcubierta finita y no\(K\) es compacta.

    Demostramos a continuación que en el espacio euclidiano dimensional finito cada conjunto delimitado cerrado es compacto. Por lo tanto, los conjuntos delimitados cerrados de\({\mathbb{R}}^n\) son ejemplos de conjuntos compactos. No es cierto que en cada espacio métrico, cerrado y acotado sea equivalente a compacto. Hay muchos espacios métricos donde cerrado y acotado no es suficiente para dar compacidad, ver por ejemplo.

    Una propiedad útil de los conjuntos compactos en un espacio métrico es que cada secuencia tiene una subsecuencia convergente. Tales conjuntos a veces se llaman secuencialmente compactos. Demostremos que en el contexto de los espacios métricos, un conjunto es compacto si y solo si es secuencialmente compacto.

    [thm:mscompactisseqcpt] Dejar\((X,d)\) ser un espacio métrico. Entonces\(K \subset X\) es un conjunto compacto si y solo si cada secuencia en\(K\) tiene una subsecuencia que converge a un punto en\(K\).

    Dejar\(K \subset X\) ser un conjunto y\(\{ x_n \}\) una secuencia en\(K\). Supongamos que para cada uno\(x \in K\), hay una pelota\(B(x,\alpha_x)\) para algunos\(\alpha_x > 0\) tal que\(x_n \in B(x,\alpha_x)\) por sólo finitamente muchos\(n \in {\mathbb{N}}\). Entonces\[K \subset \bigcup_{x \in K} B(x,\alpha_x) .\] Cualquier colección finita de estas bolas va a contener sólo finitamente muchas\(x_n\). Así para cualquier colección finita de tales bolas hay una\(x_n \in K\) que no está en la unión. Por lo tanto, no\(K\) es compacto.

    Entonces si\(K\) es compacto, entonces existe un\(x \in K\) tal que para cualquiera\(\delta > 0\),\(B(x,\delta)\) contiene\(x_k\) para infinitamente muchos\(k \in {\mathbb{N}}\). \(B(x,1)\)contiene algunos\(x_k\) así que vamos\(n_1 := k\). Si\(n_{j-1}\) se define, entonces debe existir\(k > n_{j-1}\) tal que\(x_k \in B(x,\nicefrac{1}{j})\), así definir\(n_j := k\). Fíjese en eso\(d(x,x_{n_j}) < \nicefrac{1}{j}\). Por,\(\lim\, x_{n_j} = x\).

    Para la otra dirección, supongamos que cada secuencia en\(K\) tiene una subsecuencia que converge en\(K\). Tome una cubierta abierta\(\{ U_\lambda \}_{\lambda \in I}\) de\(K\). Para cada\(x \in K\), definir\[\delta(x) := \sup \{ \delta \in (0,1) : B(x,\delta) \subset U_\lambda \text{ for some } \lambda \in I \} .\] As\(\{ U_\lambda \}\) es una cubierta abierta de\(K\),\(\delta(x) > 0\) para cada uno\(x \in K\). Por construcción, para cualquier positivo debe\(\epsilon < \delta(x)\) existir\(\lambda \in I\) tal que\(B(x,\epsilon) \subset U_\lambda\).

    Escoge un\(\lambda_0 \in I\) y mira\(U_{\lambda_0}\). Si\(K \subset U_{\lambda_0}\), nos detenemos ya que hemos encontrado una subcubierta finita. De lo contrario, debe haber un punto\(x_1 \in K \setminus U_{\lambda_0}\). Debe existir alguna\(\lambda_1 \in I\) tal que\(x_1 \in U_{\lambda_1}\) y de hecho\(B\bigl(x_1,\frac{1}{2}\delta({x_1})\bigr) \subset U_{\lambda_1}\). Trabajamos inductivamente. Supongamos que\(\lambda_{n-1}\) se define. O\(U_{\lambda_0} \cup U_{\lambda_1} \cup \cdots \cup U_{\lambda_{n-1}}\) es una cubierta finita de\(K\), en cuyo caso nos detenemos, o debe haber un punto\(x_n \in K \setminus \bigl( U_{\lambda_1} \cup U_{\lambda_2} \cup \cdots \cup U_{\lambda_{n-1}}\bigr)\). En este caso, debe haber alguna\(\lambda_n \in I\) tal que\(x_n \in U_{\lambda_n}\), y de hecho\[B\bigl(x_n,\tfrac{1}{2}\delta(x_n)\bigr) \subset U_{\lambda_n}.\]

    Entonces, o obtuvimos una subcubierta finita o obtuvimos una secuencia infinita\(\{ x_n \}\) como la anterior. Por contradicción supongamos que no hubo subcobertura finita y tenemos la secuencia\(\{ x_n \}\). Entonces hay una subsecuencia\(\{ x_{n_k} \}\) que converge, es decir,\(x = \lim \, x_{n_k} \in K\). Tomamos\(\lambda \in I\) tal que\(B\bigl(x,\frac{1}{2}\delta(x)\bigr) \subset U_\lambda\). A medida que la subsecuencia converge, hay\(k\) tal que\(d(x_{n_k},x) < \frac{1}{8}\delta(x)\). Por la desigualdad triángulo,\(B\bigl(x_{n_k},\frac{3}{8}\delta(x)\bigr) \subset B\bigl(x,\frac{1}{2}\delta(x)\bigr) \subset U_\lambda\). Entonces\(\frac{3}{8}\delta(x) < \delta({x_{n_k}})\), lo que implica\[B\bigl(x_{n_k},\tfrac{3}{16}\delta(x)\bigr) \subset B\bigl(x_{n_k},\tfrac{1}{2}\delta(x_{n_k})\bigr) \subset U_{\lambda_{n_k}}.\] As\(\nicefrac{1}{8} < \nicefrac{3}{16}\), tenemos\(x \in B\bigl(x_{n_k},\frac{3}{16}\delta(x)\bigr)\), o\(x \in U_{\lambda_{n_k}}\). Como\(\lim x_{n_j} = x\), para todos lo suficientemente\(j\) grandes tenemos\(x_{n_j} \in U_{\lambda_{n_k}}\) por. Arreglemos uno de esos\(j\) tales que\(j > k\). Pero por construcción\(x_{n_j} \notin U_{\lambda_{n_k}}\) si\(j > k\), que es una contradicción.

    Por el teorema de Bolzano-Weierstrass para secuencias () tenemos que cualquier secuencia acotada tiene una subsecuencia convergente. Por lo tanto, cualquier secuencia en un intervalo cerrado\([a,b] \subset {\mathbb{R}}\) tiene una subsecuencia convergente. El límite también debe estar en la\([a,b]\) medida en que los límites preservan las desigualdades no estrictas. Por lo tanto, un intervalo delimitado cerrado\([a,b] \subset {\mathbb{R}}\) es compacto.

    Dejar\((X,d)\) ser un espacio métrico y dejar\(K \subset X\) ser compacto. Supongamos que\(E \subset K\) es un conjunto cerrado, entonces\(E\) es compacto.

    Dejar\(\{ x_n \}\) ser una secuencia en\(E\). También es una secuencia en\(K\). Por lo tanto, tiene una subsecuencia convergente\(\{ x_{n_j} \}\) que converge a\(x \in K\). Como\(E\) se cierra el límite de una secuencia en también\(E\) está adentro\(E\) y así\(x \in E\). Por lo tanto,\(E\) debe ser compacto.

    [thm:msbw] Un subconjunto delimitado cerrado\(K \subset {\mathbb{R}}^n\) es compacto.

    Porque\({\mathbb{R}}= {\mathbb{R}}^1\) si\(K \subset {\mathbb{R}}\) está cerrado y acotado, entonces cualquier secuencia\(\{ x_n \}\) en\(K\) está delimitada, por lo que tiene una subsecuencia convergente por el teorema de Bolzano-Weierstrass para las secuencias (). Como\(K\) está cerrado, el límite de la subsecuencia debe ser un elemento de\(K\). Así\(K\) es compacto.

    Llevemos a cabo la prueba para\(n=2\) y dejemos arbitrario\(n\) como ejercicio.

    Como\(K\) está acotado, existe un conjunto\(B=[a,b]\times[c,d] \subset {\mathbb{R}}^2\) tal que\(K \subset B\). Si podemos demostrar que\(B\) es compacto, entonces\(K\), al ser un subconjunto cerrado de un compacto\(B\), también es compacto.

    Dejar\(\{ (x_k,y_k) \}_{k=1}^\infty\) ser una secuencia en\(B\). Es decir,\(a \leq x_k \leq b\) y\(c \leq y_k \leq d\) para todos\(k\). Una secuencia acotada tiene una subsecuencia convergente así que hay una subsecuencia\(\{ x_{k_j} \}_{j=1}^\infty\) que es convergente. La subsecuencia también\(\{ y_{k_j} \}_{j=1}^\infty\) es una secuencia acotada por lo que existe una subsecuencia\(\{ y_{k_{j_i}} \}_{i=1}^\infty\) que es convergente. Una subsecuencia de una secuencia convergente sigue siendo convergente, así\(\{ x_{k_{j_i}} \}_{i=1}^\infty\) es convergente. Let\[x := \lim_{i\to\infty} x_{k_{j_i}} \qquad \text{and} \qquad y := \lim_{i\to\infty} y_{k_{j_i}} .\] By,\(\bigl\{ (x_{k_{j_i}},y_{k_{j_i}}) \bigr\}_{i=1}^\infty\) converge a\((x,y)\) lo que\(i\) va a\(\infty\). Además, como\(a \leq x_k \leq b\) y\(c \leq y_k \leq d\) para todos\(k\), eso lo sabemos\((x,y) \in B\).

    Ejercicios

    Dejar\((X,d)\) ser un espacio métrico y\(A\) un subconjunto finito de\(X\). Demostrar que\(A\) es compacto.

    \(A = \{ \nicefrac{1}{n} : n \in {\mathbb{N}}\} \subset {\mathbb{R}}\)Dejar. a) Mostrar que no\(A\) es compacto directamente usando la definición. b) Mostrar que\(A \cup \{ 0 \}\) es compacto directamente usando la definición.

    Dejar\((X,d)\) ser un espacio métrico con el discreto métrico. a) Probar que\(X\) está completo. b) Probar que\(X\) es compacto si y solo si\(X\) es un conjunto finito.

    a) Demostrar que la unión de finitamente muchos conjuntos compactos es un conjunto compacto. b) Encuentre un ejemplo donde la unión de infinitamente muchos conjuntos compactos no sea compacta.

    Demostrar por dimensión arbitraria. Pista: El truco es usar la notación correcta.

    Demuestre que un conjunto compacto\(K\) es un espacio métrico completo.

    Dejar\(C([a,b])\) ser el espacio métrico como en. Mostrar que\(C([a,b])\) es un espacio métrico completo.

    [exercise:msclbounnotcompt] Dejar\(C([0,1])\) ser el espacio métrico de. Dejar\(0\) denotar la función cero. Después demuestre que la bola cerrada no\(C(0,1)\) es compacta (aunque esté cerrada y acotada). Consejos: Construir una secuencia de distintas funciones continuas\(\{ f_n \}\) tales que\(d(f_n,0) = 1\) y\(d(f_n,f_k) = 1\) para todos\(n \not= k\). Demuestre que el conjunto\(\{ f_n : n \in {\mathbb{N}}\} \subset C(0,1)\) está cerrado pero no compacto. Ver para inspirarse.

    Demuestre que existe una métrica en\({\mathbb{R}}\) que\({\mathbb{R}}\) se convierte en un conjunto compacto.

    Supongamos que\((X,d)\) está completo y supongamos que tenemos una colección contablemente infinita de conjuntos compactos no vacíos\(E_1 \supset E_2 \supset E_3 \supset \cdots\) luego probar\(\bigcap_{j=1}^\infty E_j \not= \emptyset\).

    Dejar\(C([0,1])\) ser el espacio métrico de. \(K\)Sea el conjunto de\(f \in C([0,1])\) tal que\(f\) sea igual a un polinomio cuadrático, es decir\(f(x) = a+bx+cx^2\), y tal que\(\left\lvert {f(x)} \right\rvert \leq 1\) para todos\(x \in [0,1]\), es decir\(f \in C(0,1)\). Demostrar que\(K\) es compacto.

    Colaboradores y Atribuciones


    This page titled 8.4: Completitud y compacidad is shared under a CC BY-SA 4.0 license and was authored, remixed, and/or curated by Jiří Lebl via source content that was edited to the style and standards of the LibreTexts platform; a detailed edit history is available upon request.